1answer.
Ask question
Login Signup
Ask question
All categories
  • English
  • Mathematics
  • Social Studies
  • Business
  • History
  • Health
  • Geography
  • Biology
  • Physics
  • Chemistry
  • Computers and Technology
  • Arts
  • World Languages
  • Spanish
  • French
  • German
  • Advanced Placement (AP)
  • SAT
  • Medicine
  • Law
  • Engineering
Setler [38]
3 years ago
8

1) 2/7×1/2 2) 5/8×4/5 3) 1/6×2/34) 1/3×3/45) 1/2×2/36) 1/3×2/5​

Mathematics
1 answer:
nignag [31]3 years ago
5 0
1) 1/7
2) 1/2
3) 1/9
4) 1/4
5) 1/3
6) 2/15
hope this helped:)
You might be interested in
WhT is the answer to -8(-4f-9)
Ne4ueva [31]

Answer:

32f+72

Step-by-step explanation:

Distribute the -8

7 0
3 years ago
Read 2 more answers
Hello, Brainly community!
ioda

Answer:

(B)  \displaystyle \frac{W(3.1) - W(2.9)}{0.2}

General Formulas and Concepts:

<u>Calculus</u>

Limits

Derivatives

  • The definition of a derivative is the slope of the tangent line.

Derivative Notation

Instantaneous Rates

  • Tangent Line: \displaystyle f'(x) = \frac{f(b) - f(a)}{b - a}

Step-by-step explanation:

Since we are trying to find a <em>rate</em> at which W(t) changes, we must find the <em>derivative</em> at <em>t</em> = 3.

We are given 2 close answer choices that would have the same <em>numerical</em> answer but different <em>meanings</em>:

  1. (A)  \displaystyle  \lim_{t \to 3} W(t)
  2. (B)  \displaystyle \frac{W(3.1) - W(2.9)}{0.2}

If we look at answer choice (A), we see that our units would simply just be volume. It would not have the units of a rate of change. Yes, it may be the closest numerically correct answer, but it does not tell us the <em>rate</em> at which the volume would be changing and it is not a derivative.

If we look at answer choice (B), we see that our units would be cm³/s, and that is most certainly a rate of change. Answer choice (B) is also a <em>derivative</em> at <em>t</em> = 3, and a derivative tells us what <em>rate</em> something is changing.

∴ Answer choice (B) will give us the best estimate for the value of the instantaneous rate of change of W(t) when <em>t</em> = 3.

Topic: AP Calculus AB/BC (Calculus I/I + II)

Unit: Differentiation

Book: College Calculus 10e

8 0
3 years ago
Determine the location and values of the absolute maximum and absolute minimum for given function : f(x)=(‐x+2)4,where 0&lt;×&lt
brilliants [131]

Answer:

Where 0 < x < 3

The location of the local minimum, is (2, 0)

The location of the local maximum is at (0, 16)

Step-by-step explanation:

The given function is f(x) = (x + 2)⁴

The range of the minimum = 0 < x < 3

At a local minimum/maximum values, we have;

f'(x) = \dfrac{(-x + 2)^4}{dx}  = -4 \cdot (-x + 2)^3 = 0

∴ (-x + 2)³ = 0

x = 2

f''(x) = \dfrac{ -4 \cdot (-x + 2)^3}{dx}  = -12 \cdot (-x + 2)^2

When x = 2, f''(2) = -12×(-2 + 2)² = 0 which gives a local minimum at x = 2

We have, f(2) = (-2 + 2)⁴ = 0

The location of the local minimum, is (2, 0)

Given that the minimum of the function is at x = 2, and the function is (-x + 2)⁴, the absolute local maximum will be at the maximum value of (-x + 2) for 0 < x < 3

When x = 0, -x + 2 = 0 + 2 = 2

Similarly, we have;

-x + 2 = 1, when x = 1

-x + 2 = 0, when x = 2

-x + 2 = -1, when x = 3

Therefore, the maximum value of -x + 2, is at x = 0 and the maximum value of the function where 0 < x < 3, is (0 + 2)⁴ = 16

The location of the local maximum is at (0, 16).

5 0
3 years ago
En una pasteleria realizan 15 posteles cada 30 min
artcher [175]

Answer:

60

Step-by-step explanation:

había multiplicado 4 y 15 espero que esto ayude

4 0
3 years ago
Please answer this correctly
Andrej [43]
I believe the answer is 120. 3 5/6 turns into 3.833 when turned into a decimal. 3.833 would round up to 4 which is the nearest whole number. 34 3/7 would turn into 34.4285 as a decimal which turns into 30 when rounded to the nearest ten.
4 0
3 years ago
Other questions:
  • What is 3x - 4 + 20 =55
    10·1 answer
  • At what point does the line given by the following equation cross the x-axis? Y= -3/7 +4
    10·2 answers
  • This semicircle has a diameter of 5 meters. What is the area of this figure? Use 3.14 to for pi. Enter your answer as a decimal
    7·2 answers
  • Please help me with this I attached the pictures!
    11·1 answer
  • Will give u a crown
    10·1 answer
  • What does the product of any whole-number factor multiplied by 100 always have ? Explain
    9·2 answers
  • What is the volume please explain your answer
    8·1 answer
  • PLEASE HELP SOON Find the value of x. Round to the nearest tenth. 27° х 34° 11 X = ? [?] 9 Law of Sines: sin A sin C sin B b a E
    12·1 answer
  • Pls help me on area of polygons
    13·1 answer
  • if i had 39 containers of water and each container held 24 quarts. How many quarts water are in the 39 containers?
    12·1 answer
Add answer
Login
Not registered? Fast signup
Signup
Login Signup
Ask question!